matheraum.de
Raum für Mathematik
Offene Informations- und Nachhilfegemeinschaft

Für Schüler, Studenten, Lehrer, Mathematik-Interessierte.
Hallo Gast!einloggen | registrieren ]
Startseite · Forum · Wissen · Kurse · Mitglieder · Team · Impressum
Forenbaum
^ Forenbaum
Status Mathe
  Status Schulmathe
    Status Primarstufe
    Status Mathe Klassen 5-7
    Status Mathe Klassen 8-10
    Status Oberstufenmathe
    Status Mathe-Wettbewerbe
    Status Sonstiges
  Status Hochschulmathe
    Status Uni-Analysis
    Status Uni-Lin. Algebra
    Status Algebra+Zahlentheo.
    Status Diskrete Mathematik
    Status Fachdidaktik
    Status Finanz+Versicherung
    Status Logik+Mengenlehre
    Status Numerik
    Status Uni-Stochastik
    Status Topologie+Geometrie
    Status Uni-Sonstiges
  Status Mathe-Vorkurse
    Status Organisatorisches
    Status Schule
    Status Universität
  Status Mathe-Software
    Status Derive
    Status DynaGeo
    Status FunkyPlot
    Status GeoGebra
    Status LaTeX
    Status Maple
    Status MathCad
    Status Mathematica
    Status Matlab
    Status Maxima
    Status MuPad
    Status Taschenrechner

Gezeigt werden alle Foren bis zur Tiefe 2

Navigation
 Startseite...
 Neuerdings beta neu
 Forum...
 vorwissen...
 vorkurse...
 Werkzeuge...
 Nachhilfevermittlung beta...
 Online-Spiele beta
 Suchen
 Verein...
 Impressum
Das Projekt
Server und Internetanbindung werden durch Spenden finanziert.
Organisiert wird das Projekt von unserem Koordinatorenteam.
Hunderte Mitglieder helfen ehrenamtlich in unseren moderierten Foren.
Anbieter der Seite ist der gemeinnützige Verein "Vorhilfe.de e.V.".
Partnerseiten
Dt. Schulen im Ausland: Mathe-Seiten:Weitere Fächer:

Open Source FunktionenplotterFunkyPlot: Kostenloser und quelloffener Funktionenplotter für Linux und andere Betriebssysteme
StartseiteMatheForenTopologie und Geometrieerstes Abz.axiom nicht erfüllt
Foren für weitere Schulfächer findest Du auf www.vorhilfe.de z.B. Deutsch • Englisch • Französisch • Latein • Spanisch • Russisch • Griechisch
Forum "Topologie und Geometrie" - erstes Abz.axiom nicht erfüllt
erstes Abz.axiom nicht erfüllt < Topologie+Geometrie < Hochschule < Mathe < Vorhilfe
Ansicht: [ geschachtelt ] | ^ Forum "Topologie und Geometrie"  | ^^ Alle Foren  | ^ Forenbaum  | Materialien

erstes Abz.axiom nicht erfüllt: Frage (beantwortet)
Status: (Frage) beantwortet Status 
Datum: 15:27 Do 05.04.2012
Autor: Herr_von_Omikron

Aufgabe
[mm] \{A_{i},i\in I\} [/mm] sei eine überabzählbare Menge von topologischen Räume, wobei [mm] |A_{i}|\geq [/mm] 2 für alle [mm] i\in [/mm] I gelte. Weiters möge die Topologie auf keinem der [mm] A_{i}, i\in [/mm] I die triviale Topologie sein.
Nun soll gezeigt werden, dass [mm] \prod\limits_{i\in I}{A_{i}} [/mm] mit der Produkttopologie das erste Abzählbarkeitsaxiom nicht erfüllt.

Meine Frage: wie zeige ich das?

Vorschlag: in jeder Menge [mm] A_{i} [/mm] existieren mindestens drei offene Mengen, [mm] O_{1}=\emptyset, O_{2}, O_{3}=A_{i}. [/mm] Daher kann ich für die Produkttopologie für jede Komponente eine Menge [mm] B_{i}, i\in [/mm] I bilden, die in der i-ten Komponente die Menge [mm] O_{2} [/mm] hat und in allen anderen Komponenten die leere Menge.
Alle diese [mm] B_{i} [/mm] sind dann auch in der Produkttopologie offen. Ich kann keine dieser [mm] B_{i} [/mm] als Vereinigung der anderen darstellen, also müssen sie in jeder Basis enthalten sein. Somit hat jede Basis überabzählbar viele Elemente und daher ist das 2. Abzählbarkeitsaxiom nicht erfüllt, woraus folgt, dass auch das erste Abzählbarkeitsaxiom nicht erfüllt ist.

Nur bin ich mit meinem Beweis überhaupt nicht zufrieden. Er erscheint mir nicht sehr richtig, und außerdem hab ich nicht verwendet, dass die Topologie nicht-trivial ist. Daher bitte ich um eure Hilfe.

Grüße,
Herr von Omikron

        
Bezug
erstes Abz.axiom nicht erfüllt: Antwort
Status: (Antwort) fertig Status 
Datum: 11:07 Fr 06.04.2012
Autor: SEcki


> Vorschlag: in jeder Menge [mm]A_{i}[/mm] existieren mindestens drei
> offene Mengen, [mm]O_{1}=\emptyset, O_{2}, O_{3}=A_{i}.[/mm] Daher
> kann ich für die Produkttopologie für jede Komponente
> eine Menge [mm]B_{i}, i\in[/mm] I bilden, die in der i-ten
> Komponente die Menge [mm]O_{2}[/mm] hat und in allen anderen
> Komponenten die leere Menge.

Solche Mengen existieren nicht wirklich - was soll das Produkt der leeren Menge mit einer nicht-leeren Menge sein? Da wird wohl wieder die leere Menge herauskommen.

SEcki


Bezug
                
Bezug
erstes Abz.axiom nicht erfüllt: Frage (beantwortet)
Status: (Frage) beantwortet Status 
Datum: 12:59 Fr 06.04.2012
Autor: Herr_von_Omikron

Hmm. Irgendeinen Tipp, wie ich das Problem dann angehen soll?

Bezug
                        
Bezug
erstes Abz.axiom nicht erfüllt: Antwort
Status: (Antwort) fertig Status 
Datum: 14:28 Sa 07.04.2012
Autor: tobit09

Hallo Herr von Omikron,

zunächst zwei Hinweise zu deinem ursprünglichen Ansatz:

Das zweite Abzählbarkeitsaxiom impliziert das erste, nicht umgekehrt. Also reicht es nicht zu zeigen, dass das zweite Abzählbarkeitsaxiom nicht erfüllt ist.

Du hast sehr wohl verwendet, dass die Topologien nicht trivial sind, indem du die Existenz einer dritten offenen Menge in den [mm] $A_i$ [/mm] neben der leeren Menge und ganz [mm] $A_i$ [/mm] verwendet hast. Diese dritte offene Menge wurde von dir mit [mm] $O_2$ [/mm] bezeichnet. Ich möchte sie im folgenden [mm] $W_i$ [/mm] nennen, um von den [mm] $W_i$ [/mm] für verschiedene [mm] $i\in [/mm] I$ reden zu können.

Dein Ansatz ist gut! Nimm nun für jedes [mm] $i\in [/mm] I$ ein [mm] $a_i\in W_i$ [/mm] und zeige, dass [mm] $a:=(a_i)_{i\in I}$ [/mm] keine abzählbare Umgebungsbasis besitzt.

Nimm im Widerspruch dazu an, [mm] $\mathcal{B}=\{U_n|n\in\IN\}$ [/mm] sei eine solche abzählbare Umgebungsbasis. Durch mögliches Verkleinern der [mm] $U_n$ [/mm] können wir annehmen, dass die [mm] $U_n$ [/mm] folgende Gestalt haben:

     [mm] $U_n=\produkt_{i\in I_n}V_{in}\times\produkt_{i\in I\setminus I_n}A_i$ [/mm] für gewisse [mm] $I_n\subset [/mm] I$ endlich und [mm] $V_{in}\subseteq A_i$ [/mm] offen.

Da [mm] $\bigcup_{n\in\IN}I_n\subset [/mm] I$ abzählbar und $I$ überabzählbar, existiert ein [mm] $i_0\in I\setminus\bigcup_{n\in\IN}I_n$. [/mm]

Betrachte nun

     [mm] $U:=W_{i_0}\times\produkt_{i\in I\setminus\{i_o\}}A_i$. [/mm]

Mach dir klar, dass U eine offene Umgebung von $a$ ist und verwende anschließend, dass [mm] $\mathcal{B}$ [/mm] eine Umgebungsbasis von $a$ ist.

So kannst du zu einem Widerspruch gelangen.


Viel Erfolg und viele Grüße
Tobias

Bezug
                                
Bezug
erstes Abz.axiom nicht erfüllt: Frage (beantwortet)
Status: (Frage) beantwortet Status 
Datum: 11:58 Mi 11.04.2012
Autor: Wellenoptik1


> Durch
> mögliches Verkleinern der [mm]U_n[/mm] können wir annehmen, dass
> die [mm]U_n[/mm] folgende Gestalt haben:
>  
> [mm]U_n=\produkt_{i\in I_n}V_{in}\times\produkt_{i\in I\setminus I_n}A_i[/mm]
> für gewisse [mm]I_n\subset I[/mm] endlich und [mm]V_{in}\subseteq A_i[/mm]
> offen.

Warum weißt du, daß die Mengen [mm]I_n[/mm] endlich sind??

Bezug
                                        
Bezug
erstes Abz.axiom nicht erfüllt: Antwort
Status: (Antwort) fertig Status 
Datum: 10:01 Do 12.04.2012
Autor: tobit09

Hallo Wellenoptik und herzlich [willkommenmr]!


> > Durch
> > mögliches Verkleinern der [mm]U_n[/mm] können wir annehmen, dass
> > die [mm]U_n[/mm] folgende Gestalt haben:
>  >  
> > [mm]U_n=\produkt_{i\in I_n}V_{in}\times\produkt_{i\in I\setminus I'}A_i[/mm]
> > für gewisse [mm]I_n\subset I[/mm] endlich und [mm]V_{in}\subseteq A_i[/mm]
> > offen.
>  
> Warum weißt du, daß die Mengen [mm]I_n[/mm] endlich sind??

Die Mengen der Form

     [mm] $V=\produkt_{i\in I'}V_{i}\times\produkt_{i\in I\setminus I'}A_i$ [/mm]

mit [mm]I'\subset I[/mm] endlich und [mm]V_{i}\subseteq A_i[/mm] offen bilden eine Basis der Produkttopologie.

Falls dir das unklar ist, poste bitte deine Definition von Produkttopologie.

(Wäre $I'$ nicht endlich, wäre obiges $V$ im Allgemeinen gar nicht offen bezüglich der Produkttopologie.)


EDIT: Hier stand Unfug.
Jedes [mm] $U_n$ [/mm] ist also Vereinigung von gewissen Mengen der obigen Form. Mindestens eine dieser Mengen der obigen Form enthält jeweils [mm] $(a_i)_{i\in I}$. [/mm] Nun ersetzen wir [mm] $U_n$ [/mm] durch eine solche Menge.


Viele Grüße
Tobias

Bezug
                                                
Bezug
erstes Abz.axiom nicht erfüllt: Frage (beantwortet)
Status: (Frage) beantwortet Status 
Datum: 12:56 Do 12.04.2012
Autor: Wellenoptik1


> Die Mengen der Form
>  
> [mm]V=\produkt_{i\in I'}V_{i}\times\produkt_{i\in I\setminus I'}A_i[/mm]
>  
> mit [mm]I'\subset I[/mm] endlich und [mm]V_{i}\subseteq A_i[/mm] offen bilden
> eine Basis der Produkttopologie.
>
> Jedes [mm]U_n[/mm] ist also Vereinigung von gewissen Mengen der
> obigen Form.

Hallo Tobias,

danke erstmal für deine Antwort. Dass die Mengen, die die Gestalt von V haben, eine Basis der Produkttopolgie sind, ist mir jetzt klar.
Warum aber ist jedes [mm]U_n[/mm] als Vereinigung von solchen "Basismengen" darstellbar?
Ich weiß, dass jedes Element der Produkttopologie als Vereinigung dieser Basismengen darstellbar ist. Demnach müsste jedes [mm]U_n[/mm] in der Produkttopologie und daher auch offen sein. Aber die [mm]U_n[/mm] sind doch nur Umgebungen und müssen daher nicht offen sein?!
Wo liegt mein Denkfehler.
Lg Wellenoptik

Bezug
                                                        
Bezug
erstes Abz.axiom nicht erfüllt: Antwort
Status: (Antwort) fertig Status 
Datum: 17:40 Do 12.04.2012
Autor: tobit09


> dass die Mengen, die die
> Gestalt von V haben, eine Basis der Produkttopolgie sind,
> ist mir jetzt klar.
>  Warum aber ist jedes [mm]U_n[/mm] als Vereinigung von solchen
> "Basismengen" darstellbar?
> Ich weiß, dass jedes Element der Produkttopologie als
> Vereinigung dieser Basismengen darstellbar ist. Demnach
> müsste jedes [mm]U_n[/mm] in der Produkttopologie und daher auch
> offen sein. Aber die [mm]U_n[/mm] sind doch nur Umgebungen und
> müssen daher nicht offen sein?!

Du hast völlig recht! Gut, dass du aufgepasst hast.


Korrektur:

Wir verkleinern die ursprünglichen [mm] $U_n$ [/mm] in zwei Schritten:

1. Da jedes [mm] $U_n$ [/mm] eine Umgebung von $a$ ist, gibt es jeweils eine offene Menge [mm] $U_n'$ [/mm] mit [mm] $a\in U_n'\subseteq U_n$. [/mm] Durch Ersetzen von [mm] $U_n$ [/mm] durch [mm] $U_n'$ [/mm] können wir ohne Beschränkung der Allgemeinheit annehmen, dass die [mm] $U_n$ [/mm] offen sind.

2. Jedes [mm] $U_n$ [/mm] ist also NUN Vereinigung von Basis-offenen Mengen. Mindestens eine dieser Basis-offenen Mengen enthält jeweils $a$. Nun ersetzen wir [mm] $U_n$ [/mm] durch eine solche Menge.

Bezug
                                                                
Bezug
erstes Abz.axiom nicht erfüllt: Lösung
Status: (Mitteilung) Reaktion unnötig Status 
Datum: 17:12 Sa 14.04.2012
Autor: Wellenoptik1

Eingabefehler: "{" und "}" müssen immer paarweise auftreten, es wurde aber ein Teil ohne Entsprechung gefunden (siehe rote Markierung)
Eingabefehler: "{" und "}" müssen immer paarweise auftreten, es wurde aber ein Teil ohne Entsprechung gefunden (siehe rote Markierung)
Eingabefehler: "{" und "}" müssen immer paarweise auftreten, es wurde aber ein Teil ohne Entsprechung gefunden (siehe rote Markierung)
Eingabefehler: "{" und "}" müssen immer paarweise auftreten, es wurde aber ein Teil ohne Entsprechung gefunden (siehe rote Markierung)
Eingabefehler: "{" und "}" müssen immer paarweise auftreten, es wurde aber ein Teil ohne Entsprechung gefunden (siehe rote Markierung)

Danke Tobias für deine Hilfe.
Für den interessierten Leser :-) hier nun die Lösung:

Für $i \in I$, $I$ überabzählbar, seien $(A_i,\mathcal{O}_i)$ topologische Räume mit $\vert A_i \vert \ge 2 $ und $\mathcal{O}_i$ eine nicht triviale Topologie.
Zeige:  $(\prod\limits_{i \in I} A_i, \mathcal{O})$, $\mathcal{O}$ die Produkttopologie, erfüllt das erste Abzählbarkeitsaxiom nicht.

$\mathcal{O}_i$ nicht trivial $\Rightarrow$ $\exists W_i \subset A_i: W_i \in\mathcal{O}_i \wedge \emptyset\not= W_i \not= A_i$
Da $W_i \not= \emptyset$ $\Rightarrow$ $\forall i \in I \exists a_i \in W_i$
Setze $a= (a_1, a_2, \ldots) = (a_i)_{i \in I}$ $\Rightarrow$ $a \in \prod\limits_{i \in I} W_i \subset \prod\limits_{i \in I} A_i$

Beweis durch Widerspruch
Ang. $a$ besitzt eine abzählbare Umgebungsbasis $\lbrace U_n\rbrace_{n \in\mathbb{N}$
Die Basis der Produkttopologie $\mathcal{O}$ sieht folgendermaßen aus: $\mathcal{B} := \lbrace \prod\limits_{i \in I} O_i | O_i \in\mathcal{O}_i, \exists F \subset I: \vert F \vert < \infty \wedge \forall i \not\in F: O_i = A_i \rbrace$
Betrachte $U_n$ aus der Umgebungsbasis von a: $\forall n \in\mathbb{N} \  \exists U'_n \text{offen}: a \in U'_n \subseteq U_n$
Das Mengensystem  $\lbrace U'_n\rbrace_{n \in\mathbb{N}$ ist eine offene Umgebungsbasis von a.
Jedes $U'_n$ ist offen und somit als Vereinigung von Basiselementen der Produkttopologie darstellbar. Da $a \in U'_n$ gibt es also ein Basiselement $B_n$, sodaß $a \in B_n$.
$B_n \in\mathcal{B}$ $\gdw$ $\exists F_n \subset I, \vert F_n \vert < \infty: B_n = \prod\limits_{i \in F_n} O_i \times \prod\limits_{i \not\in F_n} A_i$
$\forall n \in\mathbb{N}: \vert F_n \vert < \infty $ $\Rightarrow$ $\bigcup\limits_{n \in\mathbb{N} F_n$ ist abzählbar
$\Rightarrow$ $\exists i_0 \in I \setminus\bigcup\limits_{n \in\mathbb{N} F_n$
Betrachte die Menge $S := W_{i_0} \times  \prod\limits_{i \not= i_0} A_i$
Offensichtlich gilt $a \in S$ und $S \in\mathcal{B}$. Außerdem ist $S$, wegen der Wahl von $i_0$, keine Obermenge der Umgebungsbasis $\lbrace U'_n\rbrace_{n \in\mathbb{N}$
$S$ ist aber eine Umgebung von $a$, weil $a \in W_1 \times W_{i_0} \times \prod\limits_{i \not= i_0, i \not= 1} A_i $, d.h.: $\exists O \in\mathcal{O}: a \in O\subseteq S$
Die Tatsache das $S$ eine Umgebung ist führt zum Widerspruch.

Bezug
                                                                        
Bezug
erstes Abz.axiom nicht erfüllt: Mitteilung
Status: (Mitteilung) Reaktion unnötig Status 
Datum: 17:54 Sa 14.04.2012
Autor: tobit09

Das sieht gut aus! [ok]

Ein paar Kleinigkeiten:

> Für [mm]i \in I[/mm], [mm]I[/mm] überabzählbar, seien [mm](A_i,\mathcal{O}_i)[/mm]
> topologische Räume mit [mm]\vert A_i \vert \ge 2[/mm] und
> [mm]\mathcal{O}_i[/mm] eine nicht triviale Topologie.
> Zeige:  [mm](\prod\limits_{i \in I} A_i, \mathcal{O})[/mm],
> [mm]\mathcal{O}[/mm] die Produkttopologie, erfüllt das erste
> Abzählbarkeitsaxiom nicht.
>  
> [mm]\mathcal{O}_i[/mm] nicht trivial [mm]\Rightarrow[/mm] [mm]\exists W_i \subset A_i: W_i \in\mathcal{O}_i \wedge \emptyset\not= W_i \not= A_i[/mm]
>  
> Da [mm]W_i \not= \emptyset[/mm] [mm]\Rightarrow[/mm] [mm]\forall i \in I \exists a_i \in W_i[/mm]
>  
> Setze [mm]a= (a_1, a_2, \ldots) = (a_i)_{i \in I}[/mm] [mm]\Rightarrow[/mm] [mm]a \in \prod\limits_{i \in I} W_i \subset \prod\limits_{i \in I} A_i[/mm]

(I ist nicht abzählbar, also kann $a$ nicht die Form [mm] $(a_1,a_2,\ldots)$ [/mm] haben.)

> Beweis durch Widerspruch
>  Ang. [mm]a[/mm] besitzt eine abzählbare Umgebungsbasis [mm]\lbrace U_n\rbrace_{n \in\mathbb{N}[/mm]
>  
> Die Eine Basis der Produkttopologie [mm]\mathcal{O}[/mm] sieht
> folgendermaßen aus: [mm]\mathcal{B} := \lbrace \prod\limits_{i \in I} O_i | O_i \in\mathcal{O}_i, \exists F \subset I: \vert F \vert < \infty \wedge \forall i \not\in F: O_i = A_i \rbrace[/mm]
>  
> Betrachte [mm]U_n[/mm] aus der Umgebungsbasis von a: [mm]\forall n \in\mathbb{N} \ \exists U'_n \text{offen}: a \in U'_n \subseteq U_n[/mm]
>  
> Das Mengensystem  [mm]\lbrace U'_n\rbrace_{n \in\mathbb{N}[/mm] ist
> eine offene Umgebungsbasis von a.
>  Jedes [mm]U'_n[/mm] ist offen und somit als Vereinigung von
> Basiselementen der Produkttopologie darstellbar. Da [mm]a \in U'_n[/mm]
> gibt es also ein Basiselement [mm]B_n[/mm], sodaß [mm]a \in B_n\red{\subseteq U_n'}[/mm].
>  [mm]B_n \in\mathcal{B}[/mm]
> [mm]\gdw[/mm] [mm]\exists F_n \subset I, \vert F_n \vert < \infty: B_n = \prod\limits_{i \in F_n} O_i \times \prod\limits_{i \not\in F_n} A_i[/mm]
>  
> [mm]\forall n \in\mathbb{N}: \vert F_n \vert < \infty[/mm]
> [mm]\Rightarrow[/mm] [mm]\bigcup\limits_{n \in\mathbb{N} F_n[/mm] ist
> abzählbar
>  [mm]\Rightarrow[/mm] [mm]\exists i_0 \in I \setminus\bigcup\limits_{n \in\mathbb{N} F_n[/mm]
>  
> Betrachte die Menge [mm]S := W_{i_0} \times \prod\limits_{i \not= i_0} A_i[/mm]
>  
> Offensichtlich gilt [mm]a \in S[/mm] und [mm]S \in\mathcal{B}[/mm]. Außerdem
> ist [mm]S[/mm], wegen der Wahl von [mm]i_0[/mm], keine Obermenge der
> Umgebungsbasis [mm]\lbrace U'_n\rbrace_{n \in\mathbb{N}[/mm]

(Gemeint: Für kein [mm] $n\in\IN$ [/mm] Obermenge von [mm] $U_n'$.) [/mm]
Hier würde ich mir eine genauere Begründung wünschen.

> [mm]S[/mm] ist aber eine Umgebung von [mm]a[/mm],
> weil [mm]a \in W_1 \times W_{i_0} \times \prod\limits_{i \not= i_0, i \not= 1} A_i [/mm],

Was meinst du mit [mm] $W_1$? [/mm]

> d.h.: [mm]\exists O \in\mathcal{O}: a \in O\subseteq S[/mm]

(Nämlich z.B. $O:=S$.)

> Die Tatsache das [mm]S[/mm] eine Umgebung ist führt zum Widerspruch.

Bezug
                                                                                
Bezug
erstes Abz.axiom nicht erfüllt: Mitteilung
Status: (Mitteilung) Reaktion unnötig Status 
Datum: 18:09 Mo 16.04.2012
Autor: Wellenoptik1

Danke für den letzten Feinschliff.
Hier kurz eine eine genauere Begründung: (der Rest sollte offensichtlich sein)

> Außerdem
> ist [mm]S[/mm], wegen der Wahl von [mm]i_0[/mm], keine Obermenge der
> Umgebungsbasis [mm]\lbrace U'_n\rbrace_{n \in\mathbb{N}[/mm]
>  
> (Gemeint: Für kein [mm]n\in\IN[/mm] Obermenge von [mm]U_n'[/mm].)
>  Hier würde ich mir eine genauere Begründung wünschen.

[mm] U'_n \not\subset S [/mm], weil:
Jedes [mm] U'_n [/mm]  ist als Vereinigung von Produktbasiselementen darstellbar.
Im Speziellen ist für jedes [mm] U'_n [/mm] jeweils eines dieser Produktbasiselementen von der Form [mm] \ldots \times A_{i_0} \times \ldots [/mm], nämlich  [mm]B_n [/mm].
Also ist auch die Vereinigung dieser Produktbasiselemente jeweils von der Form  [mm] \ldots \times A_{i_0} \times \ldots [/mm] und somit auch jedes  [mm] U'_n [/mm].
Da nun aber  [mm] W_{i_0} \subset A_{i_0} [/mm] ist, kann S Obermenge von  [mm] U'_n [/mm] sein.

Bezug
                                                                                        
Bezug
erstes Abz.axiom nicht erfüllt: Mitteilung
Status: (Mitteilung) Reaktion unnötig Status 
Datum: 20:59 Mo 16.04.2012
Autor: tobit09


> [mm]U'_n \not\subset S [/mm], weil:
>  Jedes [mm]U'_n[/mm]  ist als Vereinigung von Produktbasiselementen
> darstellbar.
>  Im Speziellen ist für jedes [mm]U'_n[/mm] jeweils eines dieser
> Produktbasiselementen bei geeigneter Wahl von der Form [mm]\ldots \times A_{i_0} \times \ldots [/mm],
> nämlich  [mm]B_n [/mm].
>  Also ist auch die Vereinigung dieser
> Produktbasiselemente jeweils von der Form  [mm]\ldots \times A_{i_0} \times \ldots[/mm]

Nein, das folgt nicht.

> und somit auch jedes  [mm]U'_n [/mm].
>  Da nun aber  [mm]W_{i_0} \subset A_{i_0}[/mm] ist,

Hier meinst du wohl [mm] $W_{i_0}\subsetneq A_{i_0}$. [/mm]

> kann S keine Obermenge von  [mm]U'_n[/mm] sein.

(Hier fehlt noch das Argument [mm] $B_n\not=\emptyset$ [/mm] (wegen [mm] $a\in B_n$), [/mm] wie man sieht, wenn man es sich genauer aufschreibt.)


Du würdest dir das Leben leichter machen, wenn du nur [mm] $S\not\supseteq B_n$ [/mm] begründen würdest (was ja wegen [mm] $B_n\subseteq U_n'$ [/mm] sowieso [mm] $S\not\supseteq U_n'$ [/mm] impliziert). Konstruiere am besten explizit ein Element von [mm] $B_n$, [/mm] das nicht in $S$ liegt.

Bezug
Ansicht: [ geschachtelt ] | ^ Forum "Topologie und Geometrie"  | ^^ Alle Foren  | ^ Forenbaum  | Materialien


^ Seitenanfang ^
www.matheraum.de
[ Startseite | Forum | Wissen | Kurse | Mitglieder | Team | Impressum ]